Strengthen with Sufficient Premise Questions - - Question 8
A person can develop or outgrow asthma at any age. In children under ten, asthma is twice as likely to develop in boy...
Replies
Mehran July 21, 2018
Hi @hbrennan14@gmail.com, thanks for your post.Let's first be sure we have fully understood the stimulus. This stimulus presents a set of facts rather than an argument. Here are the facts given:
(1) a person can develop or outgrow asthma at any age
(2) in children under 10, asthma is twice as likely to develop boys
(3) boys are less likely than girls to outgrow asthma (which tells you that once a boy has developed asthma, he's less likely to outgrow it)
(4) YET by adolescence the *PERCENTAGE* of boys with asthma is about the same as the *PERCENTAGE* of girls with asthma, because
(5) a large number of girls develop asthma in early adolescence
The question stem asks you to identify the additional piece of factual information you would need to conclude that the NUMBER of adolescent boys with asthma is approximately equal to the NUMBER of adolescent girls with asthma.
In order to know something about NUMBERS based on facts about PERCENTAGES, you have to know the "size of the pie." If you had a 16" pie that yields 8 slices, and you were told to save 50% of that pie, you would know you had to save 4 slices. Right?
So the piece of information you need to conclude that the NUMBER of adolescent boys with asthma is approximately equal to the NUMBER of adolescent girls with asthma, you would need to know (C) - that there are approximately equal numbers of adolescent boys and girls in the population.
If that were *not* the case, you wouldn't know anything about the numbers of boys vs. girls based just on a fact about percentages.
Hope this is helpful. Please let us know if you have any additional questions.
JCarter January 8, 2019
Can you help me understand why E would not be correct?
Ravi January 13, 2019
@JCarter,Happy to help. In the stimulus, we're told that in children under ten,
asthma is twice as likely to develop in boys. We're also told that
boys are less likely than girls to outgrow asthma, but by adolescence,
the percentage of boys with asthma is about the same as the percentage
of girls with asthma because a large number of girls develop asthma in
early adolescence.
The question asks us to choose an answer that, if one knows, can allow
us to conclude that the number of adolescent boys with asthma is
approximately equal to the number of adolescent girls with asthma.
In the stimulus, we were given percentages, but the question is asking
us to take help draw a conclusion about absolute totals (numbers) of
adolescent girls and boys with asthma. What's missing? Well, let's
look at an example to see.
Adolescent Boys
Number: 100
Percentage with asthma: 50%
Number with asthma: 50
Adolescent Girls
Number: 200
Percentage with asthma: 50%
Number with asthma: 100
In this example, we satisfy what the stimulus says about the
percentage of adolescent boys and girls who have asthma being the
same, but the number of adolescent boys and girls with asthma is far
different, so we obviously can't conclude that the number of
adolescent boys with asthma is approximately equal to the number of
adolescent girls with asthma. What would help us conclude this? Well,
if there were equal numbers of adolescent boys and girls, and an equal
percentage of each population had asthma, then we could conclude that
the number of adolescent boys with asthma is approximately equal to
the number of adolescent girls with asthma.
This is why answer choice C is correct, as it states that there are
approximately equal numbers of adolescent boys and adolescent girls in
the population.
Answer E is incorrect because the stimulus makes no mention of the
percentage of adults with asthma. This information is irrelevant
because it would not help us make the conclusion that the number of
adolescent boys with asthma is approximately equal to the number of
adolescent girls with asthma.
Does this make sense? Let us know if you have any more questions!
wills February 25, 2019
Can you explain why B is incorrect? I was able to get to B and C and I can figure out why to choose the right answer, but I cannot figure out why not to choose the wrong answer.
Ravi February 27, 2019
@Will-Wester,Happy to help. Let's take a look.
(B) says, "children who develop asthma before two years of age are
unlikely to outgrow it"
The problem with (B) is that it does not address the issue of the
number of teens and the similar percentage of boys and girls who have
asthma, and this is what the conclusion of the argument is all about.
If (B) were true, then so what? This answer choice is not relevant to
the comparison we are looking to make between the number of teenage
boys and girls with asthma. Because of this, we can get rid of (B).
Adding (B) into the argument does not allow us to conclude that the
number of adolescent boys with asthma is approximately equal to the
number of adolescent girls with asthma. (C), on the other hand, gets
us to this conclusion, as discussed above.
Does this make sense? Let us know if you have any other questions!